Bounds on the location of the maximum Stirling numbers of the second kind (Q1043998)

From MaRDI portal
scientific article
Language Label Description Also known as
English
Bounds on the location of the maximum Stirling numbers of the second kind
scientific article

    Statements

    Bounds on the location of the maximum Stirling numbers of the second kind (English)
    0 references
    0 references
    10 December 2009
    0 references
    Let \(S(n, k)\) denote the Stirling number of the second kind. According to \textit{L. H. Harper} [Ann. Math. Stat. 38, 410--414 (1967; Zbl 0154.43703)] and by means of Newton's inequalities, the sequence \(S(n, k)\), \(k= 1,\dots,n\), is unimodal, and \(K_n\) \((1\leq K_n\leq n)\) is its unique mode if \(S(n, K_n)\neq S(n, K_n+ 1)\). Determining the value of \(K_n\) is an old problem, and a related long-standing conjecture is that there exists no \(n> 2\) such that \(S(n, K_n)= S(n, K_n+ 1)\). In particular, \textit{E. R. Canfield} and \textit{C. Pomerance} [Integers 2, Paper A01, 13 p., electronic only (2002; Zbl 1008.11006); Corrigentum: Integers 5, No. 1, Paper A09, 1 p., electronic only (2005; Zbl 1121.11305)] noted that \([e^{w(n)}]- 1\leq K_n\leq [e^{w(n)}]\) for both \(2\leq n\leq 1200\) and \(n\) large enough, where \([x]\) denotes the integer part of \(x\) and \(w(n)\) is defined by \(n= w(n)e^{w(n)}\). Using probabilistic arguments, the present author shows that \([e^{w(n)}]- 2\leq K_n\leq [e^{w(n)}]+ 1\) for \(n\geq 2\).
    0 references
    0 references
    0 references
    0 references
    0 references
    Darroch's rule
    0 references
    mode of random variable
    0 references
    Stirling number
    0 references
    probabilistic unimodal sequence
    0 references
    0 references
    0 references